Đến nội dung

Lam Ba Thinh nội dung

Có 67 mục bởi Lam Ba Thinh (Tìm giới hạn từ 04-06-2020)



Sắp theo                Sắp xếp  

#521022 $\boxed{\text{Chuyên Đề}}$ Bất đẳng thức - Cực trị

Đã gửi bởi Lam Ba Thinh on 24-08-2014 - 12:07 trong Bất đẳng thức và cực trị

202) Ta có: $\prod \left ( a+\frac{1}{a+1} \right )=\prod \left ( \frac{a^{2}}{a+1}+1 \right )=\prod \left (\frac{a^{2}}{a+1}+\frac{a+1}{4}-\frac{a}{4}+\frac{3}{4} \right )\geq \prod \left( a - \frac{a}{4}+\frac{3}{4}\right )=\frac{27}{64}\prod \left (a+1 \right )\geq \frac{27}{64}\cdot 8\prod \left (\sqrt{a} \right )\geq \frac{27}{8}$

Dấu "=" xảy ra khi và chỉ khi a=b=c=1




#521042 min A=$\frac{2}{ab}+\frac{1}...

Đã gửi bởi Lam Ba Thinh on 24-08-2014 - 15:18 trong Bất đẳng thức và cực trị

1/Tim min A=$\frac{2}{ab}+\frac{1}{a^2+b^2}+\frac{a^4+b^4}{2}$

2/cho a,b>0 va a+b<1 Tim min B=$\frac{a^2}{1-b}+\frac{b^2}{1-a}+\frac{1}{a+b}+a+b$ 

$\frac{a^{2}}{1-b}+\frac{b^{2}}{1-a}+\frac{1}{a+b}+a+b=(\frac{a^{2}}{1-b}+(1-b)\cdot \frac{1}{4})+(\frac{b^{2}}{1-a}+(1-a)\cdot \frac{1}{4})+\frac{1}{a+b}+\frac{5}{4}\cdot (a+b)-\frac{1}{2}\geq a+b+\frac{5}{4}\cdot (a+b)+\frac{1}{a+b}-\frac{1}{2}=\frac{9}{4}\cdot (a+b)+\frac{1}{a+b}-\frac{1}{2}\geq 3-\frac{1}{2}=\frac{5}{2}$

Dấu "=" xảy ra khi và chỉ khi $a=b=\frac{1}{3}$




#521563 TÌm GTNN : $x^3+y^3+xy$

Đã gửi bởi Lam Ba Thinh on 27-08-2014 - 21:30 trong Bất đẳng thức và cực trị

Ta có:

$(a-1)(a-2)\leq 0\Leftrightarrow a^{2}+2\leq 3a\Rightarrow a+\frac{2}{a}\leq 3$

Làm tương tự cho $b,c$

Cộng hết vào ta có:

$\sum a+\sum \frac{2}{a}\leq 9$

Mặt khác: $9^{2}\geq (\sum a+\sum \frac{2}{a})^{2}\geq 4\sum a.\sum \frac{2}{a}\Rightarrow 9^{2}\geq 8\sum a.\sum \frac{1}{a}\Rightarrow \sum a.\sum \frac{1}{a}\leq \frac{81}{8}$

Em xem lại đi!!!!!!

Mình nghĩ bạn mới xem lại vì bạn chưa xét đến dấu "=" xảy ra khi nào mà?




#521588 giải phương trình sau $x^{3}+\sqrt{(1-x^{2...

Đã gửi bởi Lam Ba Thinh on 27-08-2014 - 22:47 trong Phương trình - hệ phương trình - bất phương trình

Đặt $\left\{\begin{matrix}x=a & & \\ \sqrt{1-x^2}=b(b\geq 0) & & \end{matrix}\right.$

Theo hệ ta có

$\left\{\begin{matrix}a^3+b^3=\sqrt{2}ab & & \\ a^2+b^2=1 & & \end{matrix}\right.$

Đây là hệ đẳng cấp nên có:$a^3+b^3=\sqrt{2}a^2b+\sqrt{2}b^2a$

Đên đây giải được rồi nhé bạn.

Cách bạn giải hình như bị sai thì phải vì nếu như giải theo bạn thì x= 0 hoặc -1 hoặc 1

Thử các x=0;1;-1 trên nhận thấy chúng không phải là nghiệm của PT. Suy ra pt vô nghiệm

Mà theo mình thì nghiệm của PT là $\sqrt{\frac{-1}{2}}$ 




#521593 giải phương trình sau $x^{3}+\sqrt{(1-x^{2...

Đã gửi bởi Lam Ba Thinh on 27-08-2014 - 23:02 trong Phương trình - hệ phương trình - bất phương trình

Sao cậu lại suy ra nghiệm như thế.Nó có 3 nhân tử mà bạn bạn xét thiếu trường hợp rồi!Nghiệm lẻ bạn à

Bạn giải rõ ra xem nào vẫn làm theo cách của ban í




#521603 $\sqrt[3]{\frac{abc+abd+acd+bcd}{4}...

Đã gửi bởi Lam Ba Thinh on 27-08-2014 - 23:38 trong Bất đẳng thức và cực trị

Sao Đề bạn ghi 1 đằng Chứng minh ghi 1 nẻo. Vậy cái nào đúng.




#524903 $\boxed{\text{Chuyên Đề}}$ Bất đẳng thức - Cực trị

Đã gửi bởi Lam Ba Thinh on 16-09-2014 - 22:25 trong Bất đẳng thức và cực trị

210)

$x^{2}+y^{2}\geq 1-xy\geq 1-\frac{x^{2}+y^{2}}{2}\Leftrightarrow \frac{3}{2}\cdot (x^{2}+y^{2})\geq 1\Leftrightarrow x^{2}+y^{2}\geq \frac{2}{3}$

Dấu "=" xảy ra khi và chỉ khi $x=y=\frac{1}{\sqrt{3}}$.



#526335 $\boxed{\text{Chuyên Đề}}$ Bất đẳng thức - Cực trị

Đã gửi bởi Lam Ba Thinh on 26-09-2014 - 22:50 trong Bất đẳng thức và cực trị

Cho a,b,c dương. CMR:

$(\frac{4a}{c+b}+1)(\frac{4b}{a+c}+1)(\frac{4c}{a+b}+1)> 25$




#526821 $\frac{x}{x^{2}+2}+\frac{y}{y^{2}+2}+\frac{z}{z^{2}+2}...

Đã gửi bởi Lam Ba Thinh on 01-10-2014 - 21:33 trong Bất đẳng thức và cực trị

$2.(\frac{x}{x^{2}+2}+\frac{y}{y^{2}+2}+\frac{z}{z^{2}+2})\leq 2.(\frac{x}{2x+1}+\frac{y}{2y+1}+\frac{z}{2z+1})=3-(\frac{1}{2x+1}+\frac{1}{2y+1}+\frac{1}{2z+1})\leq 3-\frac{9}{2(x+y+z)+3}\leq 3-\frac{9}{2.3.\sqrt[3]{xzy}+3}=3-\frac{9}{2.3.1+3}=2\Leftrightarrow \frac{x}{x^{2}+2}+\frac{y}{y^{2}+2}+\frac{z}{z^{2}+2}\leq 1$




#527044 $\boxed{\text{Chuyên Đề}}$ Bất đẳng thức - Cực trị

Đã gửi bởi Lam Ba Thinh on 03-10-2014 - 21:11 trong Bất đẳng thức và cực trị

Cho các số thực dương $a,b,c$ .CMR:

$1) (\sum ab)(\sum \frac{1}{(a+b)^{2}})\geq \frac{9}{4}$

$2) \sum \frac{ab-bc+ca}{b^{2}+b^{2}}\geq \frac{3}{2}$

$3)(a^{2}+b^{2}+c^{2})(\frac{a}{b}+\frac{b}{c}+\frac{c}{a})\geq 4(a^{2}+b^{2}+c^{2})-(ab+bc+ca)$

$4)\sum \frac{a(b+c)}{b^{2}+c^{2}}\geq 2+\frac{8a^{2}b^{2}c^{2}}{(a^2+b^2)(b^2+c^2)(c^2+a^2)}$

$5)(a+b+c)(\frac{a}{b^2+c^2}+\frac{b}{c^2+a^2}+\frac{c}{a^2+b^2})\geq 4+\frac{4a^{2}b^{2}c^{2}}{(a^2+b^2)(b^2+c^2)(c^2+a^2)}$

$6)\frac{a^2}{b}+\frac{b^2}{c}+\frac{c^2}{a}\geq \frac{3(a^3+b^3+c^3)}{a^2+b^2+c^2}$




#527065 Sách Tuyển tập 200 bài thi vô địch toán

Đã gửi bởi Lam Ba Thinh on 03-10-2014 - 22:26 trong Tài nguyên Olympic toán

Hiện giờ em đang rất cần gấp toàn tập sách Tuyển tập 200 bài thi vô địch toán. Mong anh(chị) nào có thì có thể cho hoặc bán lại cho em. Cảm ơn mọi người nhiều.




#527075 $\frac{1}{6a+1}+\frac{1}{6b...

Đã gửi bởi Lam Ba Thinh on 03-10-2014 - 22:57 trong Bất đẳng thức và cực trị

  BĐT $\Leftrightarrow \sum \frac{6a}{6a+1}\geq \frac{18}{7}$.

 Dùng UCT ta cm được $\frac{6a}{6a+1}\geq \frac{6}{7}+\frac{162}{49}(\frac{1}{2a+1}-\frac{1}{3})\rightarrow \sum \frac{6a}{6a+1}\geq \frac{18}{7}\rightarrow Q.E.D$

   Dấu "=" xảy ra khi $a=b=c=1$.

P/s: Các bạn like ủng hộ mình nha...

Sai rồi nha bạn. Chỗ này bị ngược dấu.




#527076 Vectơ

Đã gửi bởi Lam Ba Thinh on 03-10-2014 - 23:03 trong Tài liệu, chuyên đề, phương pháp về Hình học

Cho minh hỏi là $\frac{a}{OA}+\frac{b}{OB}=1$ hay aOA+bOB=1.




#527431 $\frac{a}{ab+1}+\frac{b}{bc+1}+\frac{c}{ac+1}\geq...

Đã gửi bởi Lam Ba Thinh on 05-10-2014 - 21:15 trong Bất đẳng thức và cực trị

 

Cách khác:

Do a+b+c=3 nên tồn tại x,y,z dương thỏa mãn: a$a=\frac{3x}{x+y+z},b=\frac{3y}{x+y+z},c=\frac{3z}{x+y+z}$

Khi đó ta có:

VT = $\sum \frac{a}{ab+1}=\sum \frac{3x(x+y+z)}{9xy+(x+y+z)^{2}}$

$=3(x+y+z)\sum \frac{x^{2}}{9x^{2}y+x(x+y+z)^{2}}\geq 3(x+y+z)\frac{(x+y+z)^{2}}{9(x^{2}y+y^{2}z+z^{2}x)+(x+y+z)^{3}}$

$=\frac{3(x+y+z)^{3}}{9(x^{2}y+y^{2}z+z^{2}x)+(x+y+z)^{3}}$

 

Ta sẽ chứng minh

$(x+y+z)^{3}\geq 9(x^{2}y+y^{2}z+z^{2}x)$

 

Thật vậy, do bất đẳng thức thuần nhất nên ta có thể chuẩn hóa x+y+z=3. Ta sẽ chứng minh $3 \geq (x^{2}y+y^{2}z+z^{2}x)$

Ta có: 

$x^{2}y+y^{2}z+z^{2}x\leq\frac{xy+y}{2}+\frac{yz+z}{2}+\frac{zx+x}{2}=\frac{xy+yz+zx+3}{2}\leq 3$
 
Vậy ta có đpcm

 

 Chỗ này chứng minh sao vậy ạ?




#527544 $(a^2+b^2+c^2)(\frac{1}{(a-b)^2}+\frac...

Đã gửi bởi Lam Ba Thinh on 06-10-2014 - 20:44 trong Bất đẳng thức và cực trị

Không mất tính tổng quát giả sử $a>b>c$.

Đặt $\left\{\begin{matrix}a-b=x & & \\ b-c=y & & \end{matrix}\right. =>a-c=x+y$

Ta có:$B=\frac{1}{2}\left [ (a-b)^2+(b-c)^2+(a-c)^2 \right ].(\frac{1}{(a-b)^2}+\frac{1}{(b-c)^2}+\frac{1}{(c-a)^2})=2+\left [ (\frac{y}{x})^2+(\frac{y}{x})^2 \right ]+(\frac{x}{y}+\frac{y}{x})+1-\frac{xy}{(x+y)^2}\geq 2+2+1-\frac{1}{4}=\frac{27}{4}$ =>đpcm

 

Đây là bài toán thi VMO!

Cho mình hỏi là bạn trình như thế nào vậy mình đọc không hiểu.




#527568 $(a^2+b^2+c^2)(\frac{1}{(a-b)^2}+\frac...

Đã gửi bởi Lam Ba Thinh on 06-10-2014 - 22:00 trong Bất đẳng thức và cực trị

Không mất tính tổng quát giả sử $a>b>c$.

Đặt $\left\{\begin{matrix}a-b=x & & \\ b-c=y & & \end{matrix}\right. =>a-c=x+y$

Ta có:$B=\frac{1}{2}\left [ (a-b)^2+(b-c)^2+(a-c)^2 \right ].(\frac{1}{(a-b)^2}+\frac{1}{(b-c)^2}+\frac{1}{(c-a)^2})$$=2+\left [ (\frac{y}{x})^2+(\frac{y}{x})^2 \right ]+(\frac{x}{y}+\frac{y}{x})+1-\frac{xy}{(x+y)^2}\geq 2+2+1-\frac{1}{4}=\frac{27}{4}$ 

 

Đây là bài toán thi VMO!

Chỗ này bạn biến đổi thế nào vậy?




#527579 $\boxed{TOPIC}$ Véc-tơ và ứng dụng

Đã gửi bởi Lam Ba Thinh on 06-10-2014 - 23:19 trong Hình học phẳng

57) Trên các cạnh AB BC CA của tam giác ABC ta lấy các điểm C1, A1,B1 sao cho AC1/C1B=BA1/A1C=CB1/B1A=k. Trên các đoạn A1B1, B1C1, C1A1, ta lấy lần lượt các điểm C2, A2, B2 sao cho A1C2/C2B1=B1A2/A2C1=C1B2/B2A1=1/k.Chứng minh rằng tồn tại một phép vị tự biến tam giác ABC thành tam giác A2B2C2.




#529757 chứng minh bất đẳng thức

Đã gửi bởi Lam Ba Thinh on 20-10-2014 - 21:58 trong Bất đẳng thức và cực trị

 

Áp dụng bất đẳng thức:

$\frac{1}{x}+\frac{1}{y}\geq \frac{4}{x+y}$

Có $\frac{1}{a+b}+\frac{1}{b+c}\geq \frac{4}{a+2b+c};\frac{1}{b+c}+\frac{1}{a+c}\geq \frac{4}{a+b+2c};\frac{1}{c+a}+\frac{1}{a+b}\geq \frac{4}{2a+b+c}$.
Ta chứng minh:$\frac{1}{2a+b+c}\geq \frac{2}{2a^2+b^2+c^2+4}\Leftrightarrow 2(a-1)^2+(b-1)^2+(c-1)^2\geq 0$
$\Rightarrow \frac{1}{2a+b+c}\geq \frac{2}{2a^2+b^2+c^2+4}=\frac{2}{a^2+7}$
Tương tự rồi cộng lại có điều phải CM.Dấu = khi $a=b=c=1$

 

Bài bạn làm đúng rồi mà viết sai nhiều quá.




#529927 Cho $a,b,c$ là các số không âm thỏa mãn: $a+b+c=1007$.Chứ...

Đã gửi bởi Lam Ba Thinh on 21-10-2014 - 22:47 trong Bất đẳng thức và cực trị

Ta có:$\sqrt{2014+\frac{(b-c)^2}{2}}=\sqrt{2.a.(a+b+c)+\frac{b^2-bc+c^2}{2}}=\sqrt{\frac{4a^2+4ab+4ac+b^2-2bc+c^2}{2}}=\sqrt{\frac{(2a+b+c)^2}{2}-$$2bc$}$\leq \frac{2a+b+c}{\sqrt{2}}$

Tương tự có:$A\leq \frac{4(a+b+c)}{\sqrt{2}}\leq 2.1007.\sqrt{2}=2014\sqrt{2}$

Dấu bằng xảy ra <=>$a=b=c=\frac{1007}{3}$

Chỗ này nè đáng lẽ ra phải có 1 số = 0 nhưng vô lí nên vì vậy ta thay dấu "$\leq $" thành dấu "$<$".




#530008 Thi chọn đội tuyển tỉnh ĐĂKLĂK năm 2014-2015 (ngày 1 &2)

Đã gửi bởi Lam Ba Thinh on 22-10-2014 - 19:19 trong Thi HSG cấp Tỉnh, Thành phố. Olympic 30-4. Đề thi và kiểm tra đội tuyển các cấp.

Lời giải :

Bài 2 : Điều kiện :$x,y,z >0$

Ta có :

$(\sqrt{xy}+\sqrt{yz}+\sqrt{xz})(\frac{1}{\sqrt{xy}}+\frac{1}{\sqrt{yz}}+\frac{1}{\sqrt{xz}}) \geq 9\\$
$\Rightarrow \sqrt{xy}+\sqrt{yz}+\sqrt{xz} \geq 1 $
Rất quen thuộc $x+y+z \geq  \sqrt{xy}+\sqrt{yz}+\sqrt{xz}$
Dấu bằng xảy ra khi $x=y=z$.Từ phương trình thư nhất suy ra $x=y=z=\frac{1}{3}$
Thay vào phương trình thứ ba thấy thỏa mãn.
Vậy $(x;y;z)=(\frac{1}{3};\frac{1}{3};\frac{1}{3})$
Bài 4: Chuẩn hóa $a+b+c=3$ khi đó :
$(*)\Leftrightarrow \frac{(3-2c)^{2}}{(3-c)^{2}+c^{2}}+\frac{(3-2b)^{2}}{(3-b)^{2}+b^{2}}+\frac{(3-2a)^{2}}{(3-a)^{2}+a^{2}}\geq \frac{3}{5} (1)$
Trước hết chứng minh BĐT sau đây là đúng :
$\frac{(3-2x)^{2}}{(3-x)^{2}+x^{2}}\geq \frac{-18}{25}x+\frac{23}{25}\\$
$\Leftrightarrow \frac{36x^{3}-54x^{2}+18}{2x^{2}-6x+9}\geq 0 (*;*)$

ta có :$2x^{2}-6x+9=2(x-\frac{3}{2})^{2}+\frac{9}{2} >0$

Xét $g(x)=36x^{3}-54x^{2}+18;g'(x)=108x^{2}-108;g'(x)=0\Leftrightarrow x=0 ;x=1$

Từ đó dễ dàng suy ra $g(x) \geq g(1)=0$

Suy ra $(*;*)$ đúng .

Áp dụng bất đẳng phụ trên, ta có :

$VT_{(1)}\geq \frac{-18}{25}(a+b+c)+3.\frac{23}{25}=\frac{3}{5}=VP_{(1)}$

Suy ra dpcm. Dấu bằng xảy ra khi $a=b=c=1$

------------------------------------------

Anh tohoproirac chắc làm hết nhỉ....

Cho mình hỏi chỗ này suy ra như thế nào vậy?




#530012 $\sqrt{2x+15}=32x^{2}+32x-20$

Đã gửi bởi Lam Ba Thinh on 22-10-2014 - 19:33 trong Phương trình - hệ phương trình - bất phương trình

Giải pt bằng phương pháp đặt ẩn phụ:

$\sqrt{2x+15}=32x^{2}+32x-20$

$\sqrt{2x+15}=32x^{2}+32x-20\Leftrightarrow \sqrt{2x+15}=8(2x+15)^2-224(2x+15)+1540$

Sau đó ta đặt $t=\sqrt{2x+15} (t\geq 0)$.Từ đó giải phương trình bậc 4.




#530037 Thi chọn đội tuyển tỉnh ĐĂKLĂK năm 2014-2015 (ngày 1 &2)

Đã gửi bởi Lam Ba Thinh on 22-10-2014 - 20:50 trong Thi HSG cấp Tỉnh, Thành phố. Olympic 30-4. Đề thi và kiểm tra đội tuyển các cấp.

Hàm $f\left ( x \right )=36x^3-54x^2+18,x\in \left ( 0;3 \right )$

.Bạn nói rõ hơn được không mình vẫn chưa hiểu rõ lắm.




#530039 bài toán về số nguyên tố, số chính phuơng

Đã gửi bởi Lam Ba Thinh on 22-10-2014 - 20:58 trong Đại số


 

 

Bài 1: Tìm số nguyên $x$ lớn nhất sao cho $A= 4^{27}+4^{100}+ 4^x$ là số chính phương

Bài 2: Tìm các số nguyên tố $a,b,c$ thỏa mãn $a^4 + b^4 =c$

Bài 3: tìm các bộ số nguyên dương $(x,y,z)$ thỏa mãn: $\frac{x+\sqrt[2013]{y}}{y+\sqrt[2013]{x}}$ thuộc Q đồng thời $x^2 + y^2 + z^2$ là số nguyên tố

Bài 4:CMR nếu a,b thuộc Z; $a^2 + b^2$ chia hết cho $P$; $P=4k + 3$; $k$ thuộc $N$ thì $a$ chia hết cho $P$, $b$ chia hết cho $P$

Bài 5: Có hay không số tự nhiên chia cho $a$ và $b$ mà $a.b=2015^{2016}$ còn tổng $a+b$ chia hết cho 2016

Bài 6: Tìm các số nguyên tố $n$ để $a=n^{2014} + 1$ là số chính phương

Bài 7: Tìm số nguyên tố $p, q$ sao cho $(2^p + 2^q)$ chia hết cho $p.q$




#530063 Cho a,b>0 và a+b=1. Tìm Min của biểu thức $S=\frac{a}...

Đã gửi bởi Lam Ba Thinh on 22-10-2014 - 22:15 trong Bất đẳng thức và cực trị

$S=\frac{a\sqrt{a}}{\sqrt{(1-a)a}}+\frac{b\sqrt{b}}{\sqrt{(1-b)b}}\geq \frac{2a\sqrt{a}}{1-a+a}+\frac{2b\sqrt{b}}{1-b+b}=2(\frac{a^{2}}{\sqrt{a}}+\frac{b^{2}}{\sqrt{b}})\geq \frac{2(a+b)^2}{\sqrt{a}+\sqrt{b}}\geq \frac{2(a+b)^2}{\sqrt{2(a+b)}}=\sqrt{2}$

Dấu đẳng thức xảy ra khi và chỉ khi $a=b=\frac{1}{2}$.




#530159 Cho a,b>0 và a+b=1. Tìm Min của biểu thức $S=\frac{a}...

Đã gửi bởi Lam Ba Thinh on 23-10-2014 - 17:50 trong Bất đẳng thức và cực trị

$\frac{2(a+b)^2}{\sqrt{a}+\sqrt{b}}\geq \frac{2(a+b)^2}{\sqrt{2(a+b)}}$  mình muốn hỏi lại phần này

Bạn sử dụng BĐT này: $2(a^2+b^2)\geq (a+b)^2$.